Electrical Engineering: Basic Laws (19 of 31) The Bridge Network

Поділитися
Вставка
  • Опубліковано 5 жов 2024
  • Visit ilectureonline.com for more math and science lectures!
    In this video I will find the 6-equations and 6-unknowns of a 5-resistor bridge network.
    Next video in this series can be seen at:
    • Electrical Engineering...

КОМЕНТАРІ • 54

  • @pietro5266
    @pietro5266 7 років тому +10

    Best basic explanation I've seen of this, after a lot of searching! Thanks very much. :)

  • @noahmutiyabo5891
    @noahmutiyabo5891 2 роки тому +6

    REALLY ENJOY YOUR HARD WORK

  • @surya8891
    @surya8891 7 років тому +12

    sir , instead of using kirchoff's junction rule , can we use current division rule to relate the branching current variables ?

  • @johnyeager6521
    @johnyeager6521 3 роки тому +4

    Mr. Van Biezen has left out one important fact. The equations he came up with are correct as far as he has gone. Mr. Van Biezen has left out one very important fact. We need one more KVL equation that is a loop that includes the voltage source. Then you can solve for the currents in the resistors. If you express the currents I4 and I5 as functions of I1, I2 and I3 then you have three equations in three unknowns.

    • @MichelvanBiezen
      @MichelvanBiezen  2 роки тому +3

      You are correct. If the wrong combination of loops (equations) are picked, a solution cannot be found. To appreciate that, it is useful for the student to experiment with that so they can see how that works.

    • @trickytricks5119
      @trickytricks5119 Рік тому +1

      yeah - it's unsolvable otherwise

  • @kamrankhankami7793
    @kamrankhankami7793 4 роки тому +1

    VERY GREAT EXPLANATION SIR, i really love to watch your videos, be HAPPY sir.

  • @juxapil4526
    @juxapil4526 3 роки тому

    This is most helpful as I could understand further

  • @Deepakyadav-rh4ze
    @Deepakyadav-rh4ze 3 роки тому

    Sir I am from India and I like you explaing us very good

  • @user-km9es9jx4x
    @user-km9es9jx4x 2 роки тому +1

    thank you for saving my life

    • @MichelvanBiezen
      @MichelvanBiezen  2 роки тому +2

      That may be a bit dramatic, but we are glad that we were able to help.

  • @g2s72
    @g2s72 2 роки тому +1

    The number of equations for Kirchoff's Current Law you can writer to solve a circuit equals the number of nodes minus 1. This circuit has 4 nodes, but you have written 4 KCL equations (the first 4). So aren't you still one equation short of being able to solve the circuit?

    • @MichelvanBiezen
      @MichelvanBiezen  2 роки тому +1

      You need an equation for each of the unknown currents. (you can use Kirchhoff's current equation to eliminate one of the unknowns).

  • @gunasekarankanagasabai6067
    @gunasekarankanagasabai6067 3 роки тому +1

    Dear Sir,
    Thank you very much for all your videos. I need your notes in ppt .

  • @lagunacorona
    @lagunacorona Рік тому +1

    One of the biggest problems in understanding these problems is that many examples have the different I sections in different sides of the diamond and it gets very confusing. So when a book problem says that r1 is always I1? r2 to I2 and so forth and so forth?

    • @MichelvanBiezen
      @MichelvanBiezen  Рік тому +1

      That is a good point. (But different authors are going to use different combinations).

    • @lagunacorona
      @lagunacorona Рік тому +1

      @@MichelvanBiezen now in wheatstone bridge circuits does current always travel and split at the very first top node? why do books say there are loops of different kinds? again very confusing.

    • @MichelvanBiezen
      @MichelvanBiezen  Рік тому +1

      Current always splits in some ratio at every circuit junction. More current will flow through the branch with less resistance. Note that bridge circuits are complicated and it is better to start with some examples of simpler circuits before attemping to use Kirchhoff's rules to solve a bridge circuit.

  • @fhudge4068
    @fhudge4068 Місяць тому

    Can I instead solve the problem by using three loop currents? The two you wrote and a third that is in the mesh between the battery and the bridge?

  • @jeffreychavey4161
    @jeffreychavey4161 7 років тому +1

    my formal understanding of math is somewhat "arrested" but conceptually I reckon that the purpose of the six equations is to calculate the unique current value of each branch (I1, I2, I3, etc ...) that would satisfy each and every one of them, hence arriving at the solution. Correct?

    • @MichelvanBiezen
      @MichelvanBiezen  7 років тому +1

      Yes. (That would be a big task, but it can be done).

  • @trickytricks5119
    @trickytricks5119 Рік тому +2

    For those who want the solution, we need to include another equation that factors in the voltage source: 10 - 5(I_1) - 4(I_4) = 0, a consequence of Kirchhoff's Voltage Law.
    If we solved the following equations simultaneously:
    -> (I_T) = (I_1) + (I_2)
    -> (I_1) = (I_3) + (I_4)
    -> (I_5) = (I_2) + (I_3)
    -> 5(I_1) - 10(I_2) + 2(I_3) = 0
    -> 4(I_4) - 2(I_3) - 6(I_5) = 0
    all are derived in the video (NOTE: I am not including: (I_T) = (I_4) + (I_5))
    and -> 10 - 5(I_1) - 4(I_4) = 0
    We obtain the following answers:
    I_T = 1.74216A
    I_1 = 1.14983A
    I_2 = 0.59233A
    I_3 = 0.08711A
    I_4 = 1.06272A
    I_5 = 0.67944A
    Note, these values also satisfy the equation we excluded: (I_T) = (I_4) + (I_5):
    1.06272A + 0.67944A = 1.74216A = I_T

    • @MichelvanBiezen
      @MichelvanBiezen  Рік тому +1

      Thanks for the input.

    • @oliverdiaz4198
      @oliverdiaz4198 Рік тому

      In 10-5(i_1)-4(i_4)=0
      Why did you only use 2 resistors if it includes the whole circuit?
      Im just confused

  • @alexmontenegro9991
    @alexmontenegro9991 6 років тому +2

    These equations will not yield a result in amperes. The best you can do (as far as I can see) is end up with a series of relationships between currents.

  • @honkencheng7065
    @honkencheng7065 3 роки тому +1

    All equations have nothing to do with 10 V of source. How come?

  • @WhisperedTales713
    @WhisperedTales713 3 роки тому +1

    thnak you for the lecture how can i simplify this circuit like you did in the previous vedio

  • @ibitorucookey-gam7503
    @ibitorucookey-gam7503 3 роки тому +1

    total current is 1.74 AMPS

  • @pentagon1733
    @pentagon1733 7 років тому +2

    if you use a matrix you cant use equation 4 because its the same as equation 1 and the determinant will be zero won't it? Plus as rickmcn1986 said, you do need an equation that includes the system's emf.

    • @juanjorbrr
      @juanjorbrr 4 роки тому +1

      I tried to get the final solution with this six ecuations with six unknowleges. Using an app in my mobile (Gauss-Jordan) there are infinite solutions. The determinant of coeficient matrix is zero. So, at least one ecuation is a linear combination of other.

  • @lokeshsowreddy4727
    @lokeshsowreddy4727 6 років тому +1

    I got I= 2.1 and Req=4.74 and I1=1.4 and I2=0.7, I3=0.93, I4=0.4667, I5=1.63
    is is right??
    Plz comment

    • @janitgjay3157
      @janitgjay3157 5 років тому +2

      no

    • @trickytricks5119
      @trickytricks5119 Рік тому

      yeah, that isn't correct - did you include another equation, factoring in the voltage source? Otherwise, no solutions are possible

    • @storyteller9876
      @storyteller9876 3 місяці тому

      Answer is wrong

  • @martinhorton6937
    @martinhorton6937 4 роки тому

    Is the total resistance 16.10 ohms using d to y formula ?

  • @makkerandy
    @makkerandy 6 років тому +1

    HI,
    in case of R1 > R2 the I3 would have flown in opposite direction, isn't it?
    thanks!

    • @MichelvanBiezen
      @MichelvanBiezen  6 років тому +3

      It also depends on the value of R4 and R5. Try a few combinations and see what happens. The best way to learn.

  • @unknown-vq1gj
    @unknown-vq1gj 5 років тому +1

    what to do if the source is located inside the diamond.

  • @jevlon8736
    @jevlon8736 2 роки тому +1

    How do you know which side I3 will flow into

    • @MichelvanBiezen
      @MichelvanBiezen  2 роки тому +1

      We assume that some current (we don't know how much) flows through each of the branches. Then we label those currents for each branch. So the current flowing through R3 is now labeled as I3. Then we use one of the circuit evaluation techniques to determine what the value of that current is. (as illustrated in the video)

    • @paulian1888
      @paulian1888 3 місяці тому

      @@MichelvanBiezen What I am assuming is that due to Ohm's Law, more current will take the lower resistance path, which is 5 ohms as opposed to 10, so the higher current node won over the lower current node (the one connected to 10 ohms) and became our assumed current direction. Could be wrong.

  • @w3stsid3balla10
    @w3stsid3balla10 5 років тому +1

    How do you know which way the current in the center flows?

    • @MichelvanBiezen
      @MichelvanBiezen  5 років тому +4

      It is usually possible to determine by looking at the size of the resistors. The smallest resistors carry the greatest current. If you can't tell then you can just guess and if you guess wrong, you'll get a negative answer for the chosen current.

    • @cyrilrelatado3874
      @cyrilrelatado3874 4 роки тому

      @Michel Van Biezen, Sir what would be the answers?? I get
      I1 = 1.15 Amps
      I2 = 0.59 Amps
      I3 = 0.09 Amps
      I4 = 1.06 Amps
      I5 = 0.68 Amps
      Total current = 1.74 Amps
      Am I correct??

    • @cyrilrelatado3874
      @cyrilrelatado3874 4 роки тому +1

      @@MichelvanBiezen @Michel Van Biezen, Sir what would be the answers?? I get
      I1 = 1.15 Amps
      I2 = 0.59 Amps
      I3 = 0.09 Amps
      I4 = 1.06 Amps
      I5 = 0.68 Amps
      Total current = 1.74 Amps
      Am I correct??

    • @Dissenter1688
      @Dissenter1688 4 роки тому

      @@cyrilrelatado3874 I obtained the same total current (1.74 Amps) when working the problem with two different methods (mesh analysis & delta-y conversion).

    • @trickytricks5119
      @trickytricks5119 Рік тому

      @@cyrilrelatado3874 yes

  • @chrisstanford3652
    @chrisstanford3652 Рік тому +1

    🤗🤗